I need help I think I have the but I might be wrong

I Need Help I Think I Have The But I Might Be Wrong

Answers

Answer 1

Answer:

D

Step-by-step explanation:

3 to 7 is about 4.


Related Questions

Principal Coleman used random samples from student

records to determine how many people live in each

student's household.

What is the mean total household of the 7th-grade

sample to the nearest tenth?

7th-Grade Total Household

What is the mean total household of the 8th-grade

sample to the nearest tenth?

5,4,4,4,4, 7, 3, 4, 2, 3,

4.4, 2, 4, 3, 8, 7, 5, 4,5

8th-Grade Total Household

7,5,5,5, 3, 4, 5, 3, 2, 8,

6,5,6,3,3,4,4,53,4

Done

Answers

Answer:

7th-grade: 4.3

8th-grade: 4.5

Step-by-step explanation:

7th-grade  and 8th-Grade samples have 20 data points. The mean is obtained adding the values of the data set and dividing this addition by the amount of values.

Mean total household of the 7th-grade  sample: (5+4+4+4+4+ 7+ 3+ 4+ 2+ 3+  4+4+ 2+ 4+ 3+ 8+ 7+ 5+ 4+5)/20 = 4.3

Mean total household of the 8th-grade  sample: (7+5+5+5+3+ 4+ 5+ 3+ 2+ 8+ 6+5+6+3+3+4+4+5+3+4)/20 = 4.5

Answer:

7th grade: C

8th grade: B

Step-by-step explanation:

100% true

Brainliest to whoever gets this correct , Describe the end behavior of the following function:

Answers

Answer:

graph starts high and ends high

MegaPhone Design Inc. designs megaphones with sport team stickers on opposite sides


of the megaphone. Estimate the percent of the surface area of the megaphone


covered by the stickers. Round to the nearest percent.

Answers

*see attachment for the megaphone being referred to here

Answer:

9%

Step-by-step explanation:

==>Given:

Conical megaphone with the following dimensions:

Slant height (s) = 2.25 ft

Diameter (D) = 1.2 ft (radius r = 0.6 ft)

π = 3.1

Area covered by sticker on one side = area of square = 6 in × 6 in = 0.5 ft × 0.5 ft = 0.25 ft²

==>Required

% of the surface of area of the conical megaphone that is covered by the sticker = area covered by the sticker ÷ surface area of megaphone × 100

==>SOLUTION

Area covered by sticker on both sides = 0.25 ft² + 0.25 ft² = 0.5 ft²

Surface area of the megaphone = surface area of a cone = πrs + πr²

S.A of megaphone = (3.14 × 0.6 × 2.25) + (3.14 × 0.6²)

S.A = 4.239 + 1.1304

S.A = 5.3694 ft²

Therefore, % of the surface of area of the conical megaphone that is covered by the sticker = 0.5 ÷ 5.3694 × 100

= 9.31202741

= 9%

Answer:

Statistics show the megaphone was a success in South Carolina.

Step-by-step explanation:

The table shows the blood pressures of 16 clinic patients. What is the interquartile range of the data?

Answers

Answer:

i need a the data

Step-by-step explanation:

HELP ASAP!! what is the value of f(-3)

Answers

Answer:

f(-3) = 4

Step-by-step explanation:

There are two ways to do it with the information given to you, you can look at the graph or use the equation.

Equation:

Since x = -3, you would put -3 in for x in the first equation which gives 4. It looks like you were on the right track but forgot that it was negative so you ended up adding 3 instead of subtracting. At least that is where I assume the 10 comes from.

Graphical:

Looking at the graph you can also find the value. You want to find the value of y at x = -3, so you look at the spot where x = -3. You will see that there are two "points" there, one open and one closed. The closed circle one means that the graph includes -3 in its x value. The open circle one means the graph contains every number until x=-3.

So to find the y value at x = -3 you want to look at the y value that corresponds to the closed circle of the graph since it is the part that includes x=-3 and you will see that y = 4 at that spot.

When vector is added to vector , the resultant is vector such that . This resultant vector has components Rx=Ax+BxRx=Ax+Bx and Ry=Ay+ByRy=Ay+By. How do you determine the direction of , the angle the vector makes with the positive x-axis?

Answers

Answer:

a. When vector is added to vector , the resultant is vector such that its magnitude and direction is proportional to the resultant magnitude and direction of the component vectors.

b. ∅ = [tex]tan^{-1}[/tex](Rx/Ry)

Step-by-step explanation:

Vectors are used to express physical quantities with magnitude and direction. Vectors can be added by the parallelogram method or the triangular method of addition. the magnitude and direction of the resultant vector is usually proportional to the magnitude and direction of the component vectors.

The direction of the angle is found as

tan∅ = Rx/Ry

∅ = [tex]tan^{-1}[/tex](Rx/Ry)

In 2013, the average Girl Scout in New York City sold 96 boxes of cookies. The leader of Troop 5078 in New York City wants to know if the scouts in her troop sold more cookies than the average in New York City. She randomly samples 50 girls in Troop 5078 and records the number of boxes of cookies sold for each girl in the sample. The troop leader finds that her Girl Scouts each sold an average of 101.1 boxes of cookies with a standard deviation of 29.3. She analyzed her data using a t-test and obtained a p-value of 0.11. What conclusion can she draw from her data?

Answers

Answer:

We want to test if the scouts in her troop sold more cookies than the average in New York City (50), the system of hypothesis would be:  

Null hypothesis:[tex]\mu \leq 50[/tex]  

Alternative hypothesis:[tex]\mu > 50[/tex]  

The statistic for this case is given by:

[tex]t=\frac{\bar X-\mu_o}{\frac{s}{\sqrt{n}}}[/tex]  (1)  

For this case we have also the p value calculated

[tex]p_v =0.11[/tex]

If we use any significance level lower than 10% we FAIL to reject the null hypothesis (pvalue>significance) and we can conclude that the true mean is not higher than the mean for New York otherwise if we use a significance level higher than 10% the conclusion would be the opposite.

Step-by-step explanation:

Information given

[tex]\bar X=101.1[/tex] represent the sample mean

[tex]s=29.3[/tex] represent the sample standard deviation

[tex]n=50[/tex] sample size  

[tex]\mu_o =50[/tex] represent the value that we want to test

t would represent the statistic

[tex]p_v[/tex] represent the p value

System of hypothesis

We want to test if the scouts in her troop sold more cookies than the average in New York City (50), the system of hypothesis would be:  

Null hypothesis:[tex]\mu \leq 50[/tex]  

Alternative hypothesis:[tex]\mu > 50[/tex]  

The statistic for this case is given by:

[tex]t=\frac{\bar X-\mu_o}{\frac{s}{\sqrt{n}}}[/tex]  (1)  

For this case we have also the p value calculated

[tex]p_v =0.11[/tex]

Conclusion

If we use any significance level lower than 10% we FAIL to reject the null hypothesis (pvalue>significance) and we can conclude that the true mean is not higher than the mean for New York otherwise if we use a significance level higher than 10% the conclusion would be the opposite.

Answer true or false to each statement concerning a confidence interval for a population mean.a) The length of a confidence interval can be determined if you know only the margin of error.b) The margin of error can be determined if you know only the length of the confidence interval.c) The confidence interval can be obtained if you know only the margin of error.d) The confidence interval can be obtained if you know only the margin of error and the sample mean.e) The margin of error can be determined if you know only the confidence level.f) The confidence level can be determined if you know only the margin of error.g) The margin of error can be determined if you know only the confidence level, population standard deviation, and sample size.h) The confidence level can be determined if you know only the margin of error, population standard deviation, and sample size.

Answers

Answer:

The true statements Include

A) The length of a confidence interval can be determined if you know only the margin of error.

B) The margin of error can be determined if you know only the length of the confidence interval.

D) The confidence interval can be obtained if you know only the margin of error and the sample mean.

G) The margin of error can be determined if you know only the confidence level, population standard deviation, and sample size.

F) The confidence level can be determined if you know only the margin of error, population standard deviation, and sample size.

The false statements include

C) The confidence interval can be obtained if you know only the margin of error.

E) The margin of error can be determined if you know only the confidence level.

F) The confidence level can be determined if you know only the margin of error.

Step-by-step explanation:

Confidence Interval for the population mean is basically an interval of range of values where the true population mean can be found with a certain level of confidence.

Mathematically,

Confidence Interval = (Sample mean) ± (Margin of error)

Margin of Error is the width of the confidence interval about the mean.

It is given mathematically as,

Margin of Error = (Critical value) × (standard Error of the mean)

The critical value is obtained using the confidence level (and sample size for t-distributions).

Standard Error of the mean = (Standard deviation)/[√(sample size)]

Standard error of the mean = σₓ = (σ/√n)

Taking each of the statements one at a time

a) The length of a confidence interval can be determined if you know only the margin of error.

According to the mathematical expression of the confidence interval, the length of the confidence interval is 2 × Margin of error. Hence, the length of the confidence interval can be truly obtained from only the margin of error.

This statement is true.

b) The margin of error can be determined if you know only the length of the confidence interval.

This is another way of phrasing statement A. The margin of error is half of the length of the confidence interval.

Hence, this statement is also true.

c) The confidence interval can be obtained if you know only the margin of error.

The confidence interval can be obtained from an expression that involves the sample mean and margin of error.

Confidence Interval = (Sample mean) ± (Margin of error)

The confidence interval cannot be obtained from just the margin of error. Hence, this statement is false.

d) The confidence interval can be obtained if you know only the margin of error and the sample mean.

Like o stated in (c) above, the confidence interval can be obtained from an expression that involves the sample mean and margin of error.

Confidence Interval = (Sample mean) ± (Margin of error)

Hence, this statement is true.

e) The margin of error can be determined if you know only the confidence level.

To obtain the margin of error, if the confidence interval isn't used, an expression involving the critical value and the standard error of the mean is used.

Margin of Error = (Critical value) × (standard Error of the mean)

The critical value is obtained using the confidence level, but the margin of error cannot be obtained from just that, we still need the standard error of the mean.

Hence, this statement is false.

f) The confidence level can be determined if you know only the margin of error.

This is another way of phrasing statement E. Hence, this statement too is false.

g) The margin of error can be determined if you know only the confidence level, population standard deviation, and sample size.

This is true as these is all that is required to obtain the margin of error.

h) The confidence level can be determined if you know only the margin of error, population standard deviation, and sample size.

This can be a bit stressful to obtain, but it is true. It is easy to see this from all the explanation from the beginning to this point.

Hope this Helps!!!

Based on an indication that mean daily car rental rates may be higher for Boston than for Dallas, a survey of eight car rental companies in Boston is taken and the sample mean car rental rate is $47, with a standard deviation of $3. Further, suppose a survey of nine car rental companies in Dallas results in a sample mean of $44 and a standard deviation of $3. Use alpha = 0.05 to test to determine whether the average daily car rental rates in Boston are significantly higher than those in Dallas. Assume car rental rates are normally distributed and the population variances are equal. The null hypothesis for this problem is ______.
a) μ1 - μ2 < 0
b) μ1 - μ2 > 0
c) μ1 - μ2 = 1
d) μ1 - μ2 ≠ 0
e) μ1 - μ2 = 0

Answers

Answer:

Step-by-step explanation:

This is a test of 2 independent groups. The population standard deviations are not known. Let μ1 be the mean daily car rental rates for Boston and μ2 be the mean daily car rental rates for Dallas.

The random variable is μ1 - μ2 = difference in the mean daily car rental rates for Boston and the mean daily car rental rates for Dallas

We would set up the hypothesis.

The null hypothesis is

H0 : μ1 = μ2 H0 : μ1 - μ2 = 0

The alternative hypothesis is

H1 : μ1 > μ2 H1 : μ1 - μ2 > 0

Since sample standard deviation is known, we would determine the test statistic by using the t test. The formula is

(x1 - x2)/√(s1²/n1 + s2²/n2)

From the information given,

μ1 = 47

μ2 = 44

s1 = 3

s2 = 3

n1 = 8

n2 = 9

t = (47 - 44)/√(3²/8 + 3²/9)

t = 1.41

The formula for determining the degree of freedom is

df = [s1²/n1 + s2²/n2]²/(1/n1 - 1)(s1²/n1)² + (1/n2 - 1)(s2²/n2)²

df = [3²/8 + 3²/9]²/[(1/8 - 1)(3²/8)² + (1/9 - 1)(3²/9)²] = 4.515625/0.28571428571

df = 16

We would determine the probability value from the t test calculator. It becomes

p value = 0.09

Since alpha, 0.05 < than the p value, 0.09, then we would fail to reject the null hypothesis.

Juanita bought a $9000.00 savings bond that pays 4.75% annual simple interest. When she cashes in the bond, she will have to pay 16% federal income tax on the earned interest. How much money will she net if she cashes in the bond after 6 years?

Answers

Answer:

$11154.6

Step-by-step explanation:

We have that the interest would be calculated as follows:  

9000 * 0.0475 = 427.5

then this for the amount of time in years:

427.5 * 6 = 2565

Now, we have to pay 16% federal interest tax. So, that would be

2565 * 0.16 = 410.4

Therefore, the money available to us after 6 years would be:

9000 + 2565 - 410.4 = 11154.6

It means that it would be $ 11154.6

If the statement 4 > 3 is true, which of the following are true about the relationship between –4 and –3? Check all that apply.
–4 < –3
The inequality changes from greater than to less than because -4 is less than -3.
–4 > –3
The greater than inequality holds true even when the opposites of 4 and 3 are used.
–4 = –3

Answers

Answer:

A AND B

Step-by-step explanation:

TRUST MEEEEE FRENZ:)

If 4 > 3 then - 4 < -3 is true because when additive opposites of numbers are used inequality get reversed.

What is an inequality?

Inequality refers to a relationship that makes a non-equal comparison between two numbers or other mathematical expressions.

Given inequality,

4 > 3

If we multiply with -1 on both sides inequality get reversed

-1 × 4 > -1 × 3

-4 < -3.

Hence, the inequality changes from greater than to less than  when opposites of 4 and 3 are used.

Learn more about inequality here:

https://brainly.com/question/30231190.

#SPJ6

what is the area of the figure composed of a parallelogram, a square, and a rectangle QUUUIIIIKKKKK!!

Answers

Answer:

126 inches

Step-by-step explanation:

The rectangle is 75 in.

The square is 16 in.

The parallelogram is 35 in.

I hope this helps :-)

A recreational equipment supplier inds that among orders that include tents, 40% also include sleeping mats. Only 5% of orders that do not include tents do include sleeping mats. Also, 20% of orders include tents. Determine the following probabilities:

a. The order includes sleeping mats.
b. The order includes a tent given it includes sleeping mats.

Answers

Answer:

  a.  12%

  b.  2/3

Step-by-step explanation:

Given data regarding orders that do and don't include tents and/or mats, you want to find the probabilities of (a) the order includes mats; (b) the order includes a tent given that it includes mats.

Two-way table

The given data is summarized in the attached two-way table. The value in blue is given in the problem statement. The values in purple are computed from values given in the problem:

  40% of the 20% of orders include a tent and mat = 8% of orders

  5% of the 80% of orders with no tent = 4% of orders include a mat

The numbers in black make the totals come out right.

a. Includes mat

The table shows that 8% +4% = 12% of all orders include sleeping mats.

  P(m) = 12%

b. Includes tent, given includes mats

8% of the 12% of orders that include mats also include a tent. The probability of an order including a tent given that it includes a mat is ...

  P(t|m) = 8%/12% = 2/3

Individuals in a group may gain protection from I predators because of the dilution effect: when o predator stocks', the larger the number of prey individuals in the group, me smaller the chance that any particular member of the group will be the victim. Foster and Treherne" tested whether the dilution effect occurred when o predatory fish attacked groups of a marine insect, ine ocean skater. A subset of their data is presented in the table, which shows the number of predator attacks (per 5 minutes) on ocean skater groups differing in size. Calculate the average number of attacks (per 5 minutes! for each group size. Do the fish predators show a clear preference for attacking small groups over large groups [or vice versa)? Explain. For each group size, convert the average that you calculated for question 1 into the average number of attacks per individual (per 5 minutes) Is there a consistent relationship between the overage number of per individual (per 5 minutes) and group size? Explain. Are these results consistent with the dilution effect?

Answers

Answer:

ffhn

Step-by-step explanation:

A grocery store sells one dozen ears of white corn for $2.40. What is the unit rate for one ear of corn?

Answers

Unit rate= $$/lbs.

2.40/12=0.2

Your unit rate is $0.20

The number of miles a motorcycle, X, will travel on one gallon of gasoline is modeled by a normal distribution with mean 44 and standard deviation 5. If Mike starts a journey with one gallon of gasoline in the motorcycle, find the probability that, without refueling, he can travel more than 50 miles. Round your answer to four decimal places.

Answers

Answer:

[tex]P(X>50)=P(\frac{X-\mu}{\sigma}>\frac{50-\mu}{\sigma})=P(Z>\frac{50-44}{5})=P(z>1.2)[/tex]

And we can find this probability using the normal standar distribution and with the complement rule we got:

[tex]P(z>1.2)=1-P(z<1.2) =1-0.8849= 0.1151[/tex]

Step-by-step explanation:

Let X the random variable that represent the number of miles a motorcycle of a population, and for this case we know the distribution for X is given by:

[tex]X \sim N(44,5)[/tex]  

Where [tex]\mu=44[/tex] and [tex]\sigma=5[/tex]

We are interested on this probability

[tex]P(X>50)[/tex]

And we can use the z score formula given by:

[tex]z=\frac{x-\mu}{\sigma}[/tex]

And using this formula we got:

[tex]P(X>50)=P(\frac{X-\mu}{\sigma}>\frac{50-\mu}{\sigma})=P(Z>\frac{50-44}{5})=P(z>1.2)[/tex]

And we can find this probability using the normal standar distribution and with the complement rule we got:

[tex]P(z>1.2)=1-P(z<1.2) =1-0.8849= 0.1151[/tex]

Let V be a vector space, and let v1, v2, v3 and v4 be linearly independent vectors in V . In parts (a) and (b) below, determine whether each of the given sets of vectors is linearly independent or linearly dependent. Prove your answers using the definition.
u1 = v1 − 7v3, u2 = v1, u3 = v3, u4 = v3 + v4.

Answers

Answer:

Step-by-step explanation:

[tex]u_3 = v_3[/tex] and  [tex]u_2 = v_1[/tex]

[tex]u_1= v_1 -7 v_3[/tex]

Replace the first line to the second line, you have [tex]u_1= u_2-7u_3[/tex]

That shows [tex]\{u_1, u_2, u_3,u_4\}[/tex]

is not linear independent.

Consider a sampling distribution with p equals 0.11 and samples of size n each. Using the appropriate​ formulas, find the mean and the standard deviation of the sampling distribution of the sample proportion. a. For a random sample of size n equals 5000. b. For a random sample of size n equals 1000. c. For a random sample of size n equals 250.

Answers

Answer:

a) Mean 0.11 and standard deviation 0.0044.

b) Mean 0.11 and standard deviation 0.0099.

c) Mean 0.11 and standard deviation 0.0198

Step-by-step explanation:

Central Limit Theorem:

For a proportion p in a sample of size n, the sampling distribution of the sample proportion will be approximately normal with mean [tex]\mu = p[/tex] and standard deviation [tex]s = \sqrt{\frac{p(1-p)}{n}}[/tex]

In this question:

[tex]p = 0.11[/tex]

a. For a random sample of size n equals 5000.

Mean:

[tex]\mu = p = 0.11[/tex]

Standard deviation:

[tex]s = \sqrt{\frac{0.11*0.89}{5000}} = 0.0044[/tex]

Mean 0.11 and standard deviation 0.0044.

b. For a random sample of size n equals 1000.

Mean:

[tex]\mu = p = 0.11[/tex]

Standard deviation:

[tex]s = \sqrt{\frac{0.11*0.89}{1000}} = 0.0044[/tex]

Mean 0.11 and standard deviation 0.0099.

c. For a random sample of size n equals 250.

Mean:

[tex]\mu = p = 0.11[/tex]

Standard deviation:

[tex]s = \sqrt{\frac{0.11*0.89}{250}} = 0.0198[/tex]

Mean 0.11 and standard deviation 0.0198

Diastolic blood pressures are assumed to follow a normal distribution with a mean of 85 and a standard deviation of 12. What proportion of people has diastolic blood pressures less than 80?

Answers

Answer:

0.3745

Step-by-step explanation:

We have to solve the problem by calculating the z-score value that has the following formula:

z <(x - m) / sd

x is the value to evaluate (<80), m is the mean (85) and the standard deviation is sd (12)

replacing:

p (x <80) = z <(80 - 85) / 12

z <-0.416, we look for this value in the normal distribution table and it corresponds to:

p (x <80) = 0.3745

Which means that the proportion of people is 0.3745

A DVD is on sale for $1.05 off. This is a 15% discount from the original price. Use an equation to find the origina
price
O $16.05
$15.75
$10.50
O $7.00

Answers

Answer:

D. $7.00

Step-by-step explanation:

I just did it.

Match the building blocks of geometry to the statement that defines it

Answers

Answer:

Step-by-step explanation:

Diagram : A visual tool representing mathematical ideas to be interpreted.

Postulate : A mathematical statement taken as a fact.

Theorem : A mathematical statement proven using postulates and definitions.

Definition : A formal statement declaring the meaning of a word.

Shianne is looking up at the flag on the top of the sailboat she is sailing on. She is sitting 16 feet from the bottom
of the flag, and her line of sight is 58° from horizontal. Her eyes are 6 feet above the bottom of the sail boat.
To the nearest foot, how high up is the flag from the bottom of the sailboat?​

Answers

Answer:

height of the flag from the bottom of the boat ≈ 32 ft (nearest foot)

Step-by-step explanation:

She is on her boat and the angle of elevation to the top of the flag is 58°. Her eyes her 6 ft above the bottom of the boat she is sailing on. The height of the flag from the bottom of the boat is the sum of the height of the flag and the remaining height from the horizontal line of sight upward to the flag top.

A right angle triangle was formed Shianne line of sight. The angle from her line of sight is 58° from the horizontal. Therefore,

the adjacent side of the triangle formed is 16 ft which is the distance from the boat to the flag.

adjacent side = 16 ft

opposite angle = ?

using tangential ratio

tan 58° = opposite/adjacent

tan 58° = h/16

cross multiply

h = 16 × 1.60033452904

h = 25.6053524647

height of the flag from the bottom of the boat = 25.6053524647  + 6 = 31.6053524647

height of the flag from the bottom of the boat ≈ 32 ft(nearest foot)

Find the value of b in the graph of y=3x+b if it is known that the graph goes through the point: M(2,1)

Answers

Answer:

-5

Step-by-step explanation:

y=3x+b

M(2,1)

1=3*2+bb=1-6b= -5

In Evan's senior class of 240 students, 85% are planning to attend college after graduation. What is the
probability that a senior is chosen at random not planning to attend college after graduation?

Answers

Answer:

For this case we want to find the probability that a senior is chosen at random not planning to attend college after graduation so then we can use the complement rule given by:

[tex] P(A') = 1-P(A)[/tex]

Where A is the vent of interest and replacing we got:

[tex] P(A') = 1-0.85= 0.15[/tex]

So then the probability that a senior is chosen at random not planning to attend college after graduation is 0.15 or 15%

Step-by-step explanation:

For this case we know that the sample size if n =240 and we also know that the probability that in the Evan's class the any student are planeed to attend collge after graduation is:

[tex] P(A) =0.85[/tex]

For this case we want to find the probability that a senior is chosen at random not planning to attend college after graduation so then we can use the complement rule given by:

[tex] P(A') = 1-P(A)[/tex]

Where A is the vent of interest and replacing we got:

[tex] P(A') = 1-0.85= 0.15[/tex]

So then the probability that a senior is chosen at random not planning to attend college after graduation is 0.15 or 15%

Pollution begins to enter a lake at time t =0 at a rate​ (in gallons per​ hour) given by the formula f(t)​, where t is the time​ (in hours). At the same​ time, a pollution filter begins to remove the pollution at a rate g (t) as long as the pollution remains in the lake.

Required:
a. How much pollution is in the lake after 12 hours?
b.Use a graphing calculator to find the time when the rate that pollution enters the lake equals the rate the pollution is removed.
c. Find the amount of pollution in the lake at the time found in part b.
d.Use a graphing calculator to find the time when all the pollution has been removed from the lake.

Answers

Answer:

a) 71.25 gallons

b) 25 hrs

c) 105 gallons

d) 47.91 hrs

Step-by-step explanation:

Solution:-

- The rate at which the pollution enter the lake is expressed by a function f ( t ) as follows:

                            [tex]f ( t ) = 10*( 1 - e^-^0^.^5^t )[/tex]

- Where, the rate at which pollution is removed from the lake via a filter is expressed by a function g ( t ):

                             [tex]g ( t ) = 0.4t[/tex]

- We will denote ( x ) as the amount of population in lake at time ( t ).

- We will set up a linear first order ODE using rate of change of pollution ( x ) in the lake at any instance ( t ):

                            [tex]\frac{dx}{dt} = ( flow _ i_n ) - ( flow _ o_u_t )[/tex]

- The respective inflow and outflow of pollution from the lake are expressed by the functions f ( t ) and g ( t ). Hence, the ODE becomes:

                           [tex]\frac{dx}{dt} = f ( t ) - g ( t )\\\\\frac{dx}{dt} = 10*( 1 - e^-^0^.^5^t ) - 0.4t\\\\[/tex]

- Separate the variables and integrate from t = 0 to t = 12 hours.

                         [tex]x = \int\limits^1^2_0 [{10*( 1 - e^-^0^.^5^t ) - 0.4t} ] \, dt \\\\x ( 12 ) = [ 10 * ( t + 2e^-^0^.^5^t ) - 0.2t^2 ] \limits^1^2_0\\\\x ( 12 ) = [ 10 * ( 12 + 2e^-^6 ) - 0.2(12)^2 ] - [ 10* ( 0 + 1 ) - 0.2(0)^2 ]\\\\x ( 12 ) = 71.25 galls[/tex]

- We will use the graphing calculator to evaluate the time ( t ) at which the rate of inflow of pollution is equal to the rate at which pollution is removed from the lake. In other words solve the following equation:

                          [tex]f ( t ) = g ( t )\\\\10*( 1 - e^-^0^.^5^t ) = 0.4t[/tex]

- Solving for the above equation the following intersection point was observed:

                          t = 25 hours

- We will perform integration of the previously expressed ODE from t = 0 to t = 25 hrs.

                           [tex]x ( 25 ) = [ 10*(t - 2e^0^.^5^t ) - 0.2t^2 ]\limits^2^5 _ 0\\\\x ( 25 ) = [ 10*(25 - 2e^1^2^.^5 ) - 0.2(25)^2 ] - [ 20 ] \\\\x ( 25 ) = 105 galls[/tex]

- We will evaluated result of the integration from time t = 0 to time t = t set the amount of pollution ( x ) equal to zero:

                          [tex]x ( t ) = [ 10*( t - 2e^-^0^.^5^t ) - 0.2t^2 ]\limits^t_0\\\\x ( t ) = 10 + 20e^-^0^.^5^t - 0.2t^2 - 20[/tex]

- Use the derived result x ( t ) and feed it into the graphing calculator and solve for x ( t ) = 0; hence,

                       10 + 20e^( -0.5*t ) - 0.2t^2 - 20 = 0

- The answer is evaluated as t = 47.91 hr

                         

Alicia drove at a constant speed and traveled 182.4 miles in 3 hours. How many miles
would Alicia travel in 11 hours at the same speed?

Answers

Answer:

[tex] V = \frac{182.4 mi}{3 hr}= 60.8 \frac{mi}{hr}[/tex]

And then we can find the distance travelled in 11 hours with this formula:

[tex] D = Vt[/tex]

And replacing we got:

[tex] D = 60.8 \frac{mi}{hr} *11 hr =668.8 mi[/tex]

So then after 11 hours she will travel 668.8 mi

Step-by-step explanation:

For this case w eknow that Alicia drove at a constant speed 182.4 mi in 3 hours. We can find the speed with this formula:

[tex] V= \frac{D}{t}[/tex]

Where V is the velocity, D the distance and t the time if we replace we got:

[tex] V = \frac{182.4 mi}{3 hr}= 60.8 \frac{mi}{hr}[/tex]

And then we can find the distance travelled in 11 hours with this formula:

[tex] D = Vt[/tex]

And replacing we got:

[tex] D = 60.8 \frac{mi}{hr} *11 hr =668.8 mi[/tex]

So then after 11 hours she will travel 668.8 mi

Ravi is 1 1/4 (mixed fraction).dinesh is 1 1/15 times as tall as Ravi .what is Dinesh's height

Answers

Dinesh is 1 1/3

First: Convert any mixed numbers to fractions.
Then your initial equation becomes: 5/4 times 16/15

Then :Applying the fractions formula for multiplication
5*16 =80
4*15. =60


Last:Simplifying 80/60, the answer is
1 1/3

Now, when you click ROLL, the simulator will keep track of the average of all rolls so far. For example, if you roll 2 and then 4, the average will be 3. ROLL sides What is the average after 10 rolls

Answers

Step-by-step explanation:

Suppose the results obtained are numbers from 1 to 10, then after 10 rolls, the average is

(1+2+3+4+5+6+7+8+9+10)/10

= 55/10

= 5.5

Answer:

5 , 3 , 3.5     C) The average would approach a set value.

Step-by-step explanation:

The first three numbers don't matter it could be anyone 1-6.

The last answer is C

State the null and alternative hypotheses to test if the population represented by Sample 1 has a mean that is 2.0 units higher than the population represented by Sample 2. Let µd be the population mean of​ matched-pair differences for Sample 1 - Sample 2. State the null and alternative hypotheses.

Pair 1 2 3 4 5 6 7
Sample1 5 6 10 3 6 7 8
Sample2 4 4 3 5 5 5 3

Answers

Answer:

Null Hypothesis, [tex]H_0[/tex] : [tex]\mu_A-\mu_B[/tex] = 2.0 units  or  [tex]\mu_D[/tex] = 2.0 units

Alternate Hypothesis, [tex]H_A[/tex] : [tex]\mu_A-\mu_B\neq[/tex] 2.0 units  or  [tex]\mu_D\neq[/tex] 2.0 units

Step-by-step explanation:

We are given the following data below and we have test if the population represented by Sample 1 has a mean that is 2.0 units higher than the population represented by Sample 2;

Pair : 1 2 3 4 5 6 7

Sample1 : 5 6 10 3 6 7 8

Sample2 : 4 4 3 5 5 5 3

Let [tex]\mu_A[/tex] = population mean represented by Sample 1

[tex]\mu_B[/tex] = population mean represented by Sample 2

[tex]\mu_D[/tex] = population mean matched-pair differences for Sample 1 - Sample 2

So, Null Hypothesis, [tex]H_0[/tex] : [tex]\mu_A-\mu_B[/tex] = 2.0 units  or  [tex]\mu_D[/tex] = 2.0 units

Alternate Hypothesis, [tex]H_A[/tex] : [tex]\mu_A-\mu_B\neq[/tex] 2.0 units  or  [tex]\mu_D\neq[/tex] 2.0 units

Here null hypothesis states that the population represented by Sample 1 has a mean that is 2.0 units higher than the population represented by Sample 2.

On the other hand, alternate hypothesis states that the population represented by Sample 1 has a mean that is not 2.0 units higher than the population represented by Sample 2.

At the airport, d airplanes flew away, and b were still there. How many more airplanes stayed at the airport than flew away?

Answers

Answer:

b-d

Step-by-step explanation:

At the airport, d airplanes flew away, and b were still there.

Number of airplanes which flew away =d Number of airplanes which stayed at the airport =b

We are required to determine how many more airplanes stayed at the airport than flew away.

To do this, we subtract the number of airplanes which flew away from the number which stayed.

Therefore:

Number of more airplanes which stayed than flew away

=Number of airplanes which stayed-Number of airplanes which flew away

= b-d

Other Questions
what is the y-intercept of y=-3x-2 PLEASE SOMEONE HELP ME WITH THIS QUESTION ANYONE!!!!In Edward's Excellent Store, everything is 25% off. You buy a jacket that is originally for $36. When you go to the cashier, you pay 6% sales tax. How much do you pay in all?A.$62.82B.$28.62C.$38.96D.$76.83 Jillian calculates that see will take 95 minutes to run 7 miles. She runs the distance in 80 minutes. What is Jillians percent error? What is the most common reaction to flowers when haveing allergies ? A solenoid is 2.50 cm in diameter and 30.0 cm long. It has 300 turns and carries 12 A. (a) Calculate the magnetic field inside the solenoid. (b) Calculate the magnetic flux through the surface of a circle of radius 1.00 cm, which is positioned perpendicular to and centered on the axis of the solenoid. (c) The perimeter of this circle is made of conducting material. The current in the solenoid uniformly goes from 12 A to 10 A in 0.001 seconds. What e.M.F. Is generated in the conducting material? (d) Now, inside the solenoid we put a bar of steel, a ferromagnetic material. Steel has got a magnetic permeability m = 40000 . If the current in the solenoid is 12 A, what's the total magnetic field in the steel bar? productivity is the combination of 7. What is the shape of an s orbital?A dumbbellB clover leatC sphericalD donut What do historian do when using the historical thinking skill of evaluating interpretations? What ratio can you use to determine the probability of a compound event?Drag words to complete the ratio. Each word may be used only once. Kendra found this relationship between the quantities in the table.1342.1505375y75225300Multiply x by 7.5 to get yWhich error did Kendra make?Kendra should have added 75 to the x-values to get the y-valuesKendra should have added 7.5 to the x-values to get the y-valuesKendra should have multiplied the x-values by 75 to get the y-valuesKendra should have multiplied the x-values by 25 to get the y-valuesHELP IM TIMED Is the following number rational or irrational?-0.323232323232 Helllllppp please asap What are the opposites of 9, 3.6, 2.25, and 9 1 2 ? Enter the answers in respective order, each separated by comma. A calendar shop has 5,624 calendars. Each rack can hold 38 calendars. How many racks does the store need to use? Match the fascist or totalitarian powers to the countries they ruled. Write a paragraph comparing the changes between Usa and russia during the cold war. You change oil every 6000 miles and drive 2000miles a month; how many times a year do youchange oil? (What do the 2 facts help you learn?) Can you think of other scandals in sports or politics that are similar to the White Sox story in any way? The nursing is caring for client recovering from intestinal surgery. What assessment findings requires immediate intervention. need help asap. Graph the line: y= -2x +3 use {-1,0,1,2} for x.